Series Comparison Test, help?

Click For Summary
The discussion revolves around the validity of various statements regarding the convergence or divergence of series using the Comparison Test. Participants evaluate six series, with some statements deemed correct and others incorrect based on their comparisons. The consensus is that statements 1, 4, and 5 are incorrect, while statements 2, 3, and 6 are correct. The reasoning provided highlights the importance of the correct inequalities in determining convergence. Overall, the participants confirm their evaluations and reasoning for each series.
ani9890
Messages
11
Reaction score
0
Series Comparison Test, URGENT help?

Each of the following statements is an attempt to show that a given series is convergent or divergent using the Comparison Test (NOT the Limit Comparison Test.) For each statement, enter C (for "correct") if the argument is valid, or enter I (for "incorrect") if any part of the argument is flawed.

1. For all n>2, ∑ 1/ (n^2−1) < 1/n^2 so converges
2. For all n> 1, ∑ arctan(n) / n^3 < pi / 2n^3 so converges
3. For all n>1, ∑ ln(n)/n^2 < 1/n^1.5 so converges
4. For all n>1, ∑ 1/nln(n) < 2/n so diverges
5. For all n>2, ∑ n/(n^3 - 8) < 2/n^2 so converges
6. For all n>2, ∑ ln(n)/n > 1/n so diverges

I believe (1) is incorrect.
(2), (3), and (6) I believe are correct.
But I can't figure out 4 and 5 ? I think 4 is incorrect but I'm not sure, and for 5 shouldn't it be compared to 1/n^2 ?

Please help!
 
Last edited:
Physics news on Phys.org


What's your reasoning behind the numbers you have answers for?
 


for (1) 1/n^2 is actually greater than 1/ (n^2−1), for (4) if it diverges it should actually be asubn > bsubn not the other way around, and (5) the bsubn should be n/n^3 which becomes 1/n^2

for the (2),(3) 1/ n^p where p>1 so it converges which is correct, and for (6) n^p, here p=1 so it diverges which is correct.
 


Your reasoning for 1 is correct (the same reasoning applies to 5; that one has the inequality in the wrong direction).
2, 3, and 4 are correct too.
 


thank you for your help,
so I just wanted to make sure, for my answers:

1. incorrect
2. correct
3. correct
4. incorrect
5. incorrect
6. correct

is this okay?
 


Looks good :smile:
 
Question: A clock's minute hand has length 4 and its hour hand has length 3. What is the distance between the tips at the moment when it is increasing most rapidly?(Putnam Exam Question) Answer: Making assumption that both the hands moves at constant angular velocities, the answer is ## \sqrt{7} .## But don't you think this assumption is somewhat doubtful and wrong?

Similar threads

  • · Replies 3 ·
Replies
3
Views
1K
  • · Replies 4 ·
Replies
4
Views
1K
  • · Replies 6 ·
Replies
6
Views
2K
  • · Replies 2 ·
Replies
2
Views
2K
  • · Replies 1 ·
Replies
1
Views
2K
  • · Replies 7 ·
Replies
7
Views
2K
  • · Replies 14 ·
Replies
14
Views
2K
  • · Replies 2 ·
Replies
2
Views
2K
Replies
14
Views
2K
Replies
8
Views
2K